Violympic toán 9

Mai Huyền My

Cho x,y,z>0 và \(\dfrac{1}{1+x}+\dfrac{1}{1+y}+\dfrac{1}{1+z}\ge2\)

Chứng minh: xyz≤\(\dfrac{1}{8}\)

Vũ Tiền Châu
14 tháng 7 2018 lúc 16:46

Ta có \(\dfrac{1}{1+x}\ge1-\dfrac{1}{1+y}+1-\dfrac{1}{1+x}=\dfrac{y}{1+y}+\dfrac{z}{1+z}\)

\(\ge2\sqrt{\dfrac{yz}{\left(y+1\right)\left(z+1\right)}}\)

Chứng minh tương tự, ta có

\(\dfrac{1}{1+y}\ge2\sqrt{\dfrac{xz}{\left(z+1\right)\left(x+1\right)}};\dfrac{1}{1+z}\ge2\sqrt{\dfrac{xy}{\left(x+1\right)\left(y+1\right)}}\)

Nhân cả 3 cua 3 BĐT cùng chiều, ta có

\(\dfrac{1}{\left(x+1\right)\left(y+1\right)\left(z+1\right)}\ge\dfrac{8xyz}{\left(x+1\right)\left(y+1\right)\left(z+1\right)}\Rightarrow xuz\le\dfrac{1}{8}\left(ĐPCM\right)\)


Các câu hỏi tương tự
yeens
Xem chi tiết
Andromeda Galaxy
Xem chi tiết
Nguyễn Lâm Ngọc
Xem chi tiết
Tiểu Bạch Kiểm
Xem chi tiết
ITACHY
Xem chi tiết
ITACHY
Xem chi tiết
Viêt Thanh Nguyễn Hoàn...
Xem chi tiết
Ba Dao Mot Thoi
Xem chi tiết
Bùi Đức Anh
Xem chi tiết